Added week 6 recitation solutions
authorW. Trevor King <wking@drexel.edu>
Fri, 31 Jul 2009 16:21:37 +0000 (12:21 -0400)
committerW. Trevor King <wking@drexel.edu>
Thu, 17 Sep 2009 16:48:38 +0000 (12:48 -0400)
latex/problems/Young_and_Freedman_12/problem25.57.tex [new file with mode: 0644]
latex/problems/Young_and_Freedman_12/problem25.72.tex [new file with mode: 0644]
latex/problems/Young_and_Freedman_12/problem26.06.tex [new file with mode: 0644]
latex/problems/Young_and_Freedman_12/problem26.21.tex [new file with mode: 0644]
latex/problems/Young_and_Freedman_12/problem26.22.tex [new file with mode: 0644]
latex/problems/Young_and_Freedman_12/problem26.48.tex [new file with mode: 0644]

diff --git a/latex/problems/Young_and_Freedman_12/problem25.57.tex b/latex/problems/Young_and_Freedman_12/problem25.57.tex
new file mode 100644 (file)
index 0000000..6c47019
--- /dev/null
@@ -0,0 +1,63 @@
+\begin{problem*}{25.57}
+An electrical conductor designed to carry large currents has a
+circular cross section $2.50\U{mm}$ in diameter and is $14.0\U{m}$
+long.  The resistance between the ends is $0.104\U{\Ohm}$.  \Part{a}
+What is the resistivity of the material?  \Part{b} If the
+electric-field magnitude in the conductor is $1.28\U{V/m}$, what is
+the total current?  \Part{c} If the material has $8.5\E{28}$ free
+electrons per cubic meter, find the average drift speed under the
+conditions of \Part{b}.
+\end{problem*}
+
+\begin{solution}
+\Part{a}
+Using the resistivity formula (Eqn.~25.10)
+\begin{align}
+  R &= \frac{\rho}{L}{A}  &
+  \rho &= \frac{RA}{L} = \frac{R\pi(d/2)^2}{L}
+     = \frac{0.104\U{\Ohm}\cdot\pi\cdot(1.25\E{-3}\U{m})^2}{14.0\U{m}}
+     = \ans{3.65\E{-8}\U{\Ohm$\cdot$}} \;.
+\end{align}
+
+\Part{b}
+Because the electric field is constant inside the conductor, the
+voltage difference across the conductor is
+\begin{equation}
+  V = EL = 17.9\U{V} \;.
+\end{equation}
+Using Ohm's law for the current through the conductor
+\begin{align}
+  V &= IR  &  I &= \frac{V}{R} = \ans{172\U{A}} \;.
+\end{align}
+
+\Part{c}
+In a time $\Delta t$, all the electrons within $v\Delta t$ of a given
+cross section will drift through that cross section.  Since we know
+the cross sectional area of the wire is $A$, that corresponds to a
+volume of
+\begin{equation}
+  \mathcal{V} = Av\Delta t \;.
+\end{equation}
+We know the density of electrons $n = 8.5\E{28}\U{e$^-$/m$^3$}$,
+so the number of electrons crossing the section is
+\begin{equation}
+  N = n\mathcal{V} = nAv\Delta t
+\end{equation}
+and the charge crossing is
+\begin{equation}
+  Q = eN = enAv\Delta t \;,
+\end{equation}
+where $e=1.60\E{-18}\U{C}$ is the charge on one electron.
+
+The current in the wire is thus
+\begin{equation}
+  I = \frac{Q}{\Delta t} = \frac{enAv\Delta t}{\Delta t} = enAv \;,
+\end{equation}
+and the drift velocity is
+\begin{equation}
+  v = \frac{I}{enA}
+    = \frac{172\U{A}}
+      {1.8\E{-18}\U{C}\cdot8.5\E{28}\U{e$^-$/m$^3$}\cdot\pi(1.25\E{-3}\U{m})^2}
+    = \ans{2.5\U{mm/s}} \;.
+\end{equation}
+\end{solution}
diff --git a/latex/problems/Young_and_Freedman_12/problem25.72.tex b/latex/problems/Young_and_Freedman_12/problem25.72.tex
new file mode 100644 (file)
index 0000000..4d3ba34
--- /dev/null
@@ -0,0 +1,31 @@
+\begin{problem*}{25.72}
+A typical cost for electric power is $12.0\text{\textcent}$ per
+kilowatt-hour.  \Part{a} Some people leave their porch light on all
+the time.  What is the yearly cost to keep a $75\U{W}$ bulb burning
+day and night?  \Part{b} Suppose your refrigerator uses $400\U{W}$ of
+power when it's running, and it runs $8$ hours a day.  What is the
+yearly cost of operating your refrigerator?
+\end{problem*}
+
+\begin{solution}
+This is just a unit conversion problem, but it's good to get a feel
+for how much a lightbulb costs.  Note that PECO power in Philly is
+currently around $15.4\text{\textcent}/kW\cdot\text{hour}$
+(\url{http://www.jea.com/services/electric/rates_quarterly.asp}).
+
+\Part{a}
+\begin{equation}
+  75\U{W}\cdot\frac{24\U{hours}}{1\U{day}}
+         \cdot\frac{365\U{days}}{1\U{year}}
+         \cdot\frac{\$0.120}{10^3\U{W$\cdot$hour}}
+    = \ans{\$78.84}
+\end{equation}
+
+\Part{b}
+\begin{equation}
+  400\U{W}\cdot\frac{8\U{hours}}{1\U{day}}
+          \cdot\frac{365\U{days}}{1\U{year}}
+          \cdot\frac{\$0.120}{10^3\U{W$\cdot$hour}}
+    = \ans{\$140.16}
+\end{equation}
+\end{solution}
diff --git a/latex/problems/Young_and_Freedman_12/problem26.06.tex b/latex/problems/Young_and_Freedman_12/problem26.06.tex
new file mode 100644 (file)
index 0000000..396766b
--- /dev/null
@@ -0,0 +1,72 @@
+\begin{problem*}{26.6}
+For the circuit shown in Fig.~26.40 both meters are idealized, the
+battery has no appeciable internal resistance, and the ammeter reads
+$1.25\U{A}$.  \Part{a} What does the voltmeter read?  \Part{b} What is
+the emf \EMF of the battery?
+\begin{center}
+\begin{verbatim}
+             25.0
++---V---+  +-/\/\/-A-+
+|       |  | 15.0    |
++-/\/\/-+--+-/\/\/---+------+
+| 45.0     | 15.0    Z 10.0 |
+|          +-/\/\/---+      |
+|       35.0      E         |
++-------/\/\/----i|---------+
+\end{verbatim}
+\end{center}
+\end{problem*}
+
+\begin{solution}
+It's always a good idea to start off by labeling things.
+\begin{center}
+\begin{verbatim}
+            R1=25.0 I1=1.25A
++---V---+  +-/\/\/--<-+
+|       |  |R2=15.0 I2|
++-/\/\/-+--+-/\/\/--<-a----------+
+| 45.0=Rv  |R3a=15.0  Z 10.0=R3b |
+|          +-/\/\/--<-+          |
+|  It   35.0=Re   E  I3          |
++--->---/\/\/----i|--------------+
+\end{verbatim}
+\end{center}
+\Part{a}
+First, we'll get a handle on the $I2$/$I2$/$I3$ situation, since
+that's our only handle on current at the moment.  Looping
+counter-clockwise from $a$ through wires 1 and 2,
+\begin{align}
+  0 &= -I_1R_1 + I_2R_2 \\
+  I_2 &= \frac{R_1}{R_2} I_1 = \frac{25.0\U{\Ohm}}{15.0\U{\Ohm}}1.25\U{A}
+    = 2.08\U{A} \;.
+\end{align}
+Looping counter-clockwise from $a$ through wires 1 and 3,
+\begin{align}
+  0 &= -I_1R_1 + I_3R_{3a} + I_3R_{3b} = -I_1R_1 + I_3(R_{3a}+R_{3b}) \\
+  I_2 &= \frac{R_1}{R_{3a}+R_{3b}} I_1
+    = \frac{25.0\U{\Ohm}}{(15.0+10.0)\U{\Ohm}}1.25\U{A}
+    = 1.25\U{A} \;.
+\end{align}
+
+Now that we know all about that little cluster, we can move out to the
+larger loop.  Using the junction rule at junction $a$
+\begin{align}
+  0 &= I_t - I_1 - I_2 - I_3 \\
+  I_t &= I_1 + I_2 + I_3 = 4.58\U{A} \;.
+\end{align}
+Knowing the current in the larger loop, we just use Ohm's law on $R_V$
+\begin{equation}
+  V = I_t R_V = \ans{206\U{V}}
+\end{equation}
+
+\Part{b}
+For this part, we do the full loop, heading counter clockwise from $a$
+and using wire 1 (although you could use any of wires 1, 2, or 3,
+since they must all have the same voltage across them)
+\begin{align}
+  0 &= -I_1R_1 - I_tR_V - I_tR_\EMF + \EMF \\
+  \EMF &= I_1R_1 + I_tR_V + I_tR_\EMF
+    = 1.25\U{A}\cdot25.0\U{\Ohm} + 206\U{V} + 4.58\U{A}\cdot35.0\U{\Ohm}
+    = \ans{398\U{V}}
+\end{align}
+\end{solution}
diff --git a/latex/problems/Young_and_Freedman_12/problem26.21.tex b/latex/problems/Young_and_Freedman_12/problem26.21.tex
new file mode 100644 (file)
index 0000000..29013b2
--- /dev/null
@@ -0,0 +1,69 @@
+\begin{problem*}{26.21}
+In the circuit shown in Fig.~26.49 find \Part{a} the current in
+resistor $R$; \Part{b} the resistance $R$; \Part{c} the unknown emf
+\EMF.  \Part{d} If the circuit is broken at point $x$, what is the
+current in resistor $R$?
+\end{problem*}
+
+\begin{nosolution}
+\begin{center}
+\begin{verbatim}
+    28.0V     R
++----|i-----/\/\/-------+
+|    E      6.00  4.00A |
++----|i--x--/\/\/---<---+
+|         3.00    6.00A |
++---------/\/\/---->----+
+\end{verbatim}
+\end{center}
+\end{nosolution}
+
+\begin{solution}
+\begin{center}
+\begin{verbatim}
+    28.0V     R    I_1
++----|i-----/\/\/---<-------+
+|    E      6.00  4.00A=I_2 |
++----|i--x--/\/\/---<-------a
+|         3.00    6.00A=I_3 |
++---------/\/\/---->--------+
+\end{verbatim}
+\end{center}
+\Part{a}
+Labeling the currents as above, we use Kirchoff's junction rule
+summing the currents entering node $a$.
+\begin{align}
+  0 &= -I_1 - I_2 + I_3  &
+  I_1 &= I_3 - I_2 = \ans{2.00\U{A}}
+\end{align}
+
+\Part{b}
+We'll use a loop rule for this part.  We still don't know \EMF in wire
+2, so lets loop around 1 and 3 counter-clockwise starting from $a$.
+\begin{align}
+  0 &= -I_1 R + 28.0\U{V} - I_3 R_3 \\
+  R &= \frac{28.0\U{V} - I_3 R_3}{I_1}
+    = \frac{28.0\U{V} - 6.00\U{A}\cdot3.00\U{\Ohm}}{2.00\U{A}}
+    = \ans{5.00\U{\Ohm}}
+\end{align}
+
+\Part{c}
+Now we'll use a loop involving wire 2, say 2 and 3 counter-clockwise
+starting from $a$.
+\begin{align}
+  0 &= - I_2 R_2 + \EMF - I_3 R_3 \\
+  \EMF &= I_2 R_2 + I_3 R_3
+    = 4.00\U{A}\cdot6.00\U{\Ohm} + 6.00\U{A}\cdot3.00\U{\Ohm}
+    = \ans{42.0\U{V}}
+\end{align}
+
+\Part{d}
+Breaking the circuit at $x$ means that $I_2\rightarrow0$ so $I_1=I_2=I$.
+Applying our same loop rule from \Part{b}
+\begin{align}
+  0 &= -I R + 28.0\U{V} - I R_3 \\
+  I &= \frac{28.0\U{V}}{R+R_3}
+    = \frac{28.0\U{V}}{8.00\U{\Ohm}}
+    = \ans{3.50\U{A}}
+\end{align}
+\end{solution}
diff --git a/latex/problems/Young_and_Freedman_12/problem26.22.tex b/latex/problems/Young_and_Freedman_12/problem26.22.tex
new file mode 100644 (file)
index 0000000..c2ec360
--- /dev/null
@@ -0,0 +1,54 @@
+\begin{problem*}{26.22}
+Find the emfs $\EMF_1$ and $\EMF_2$ in the circuit of Fig.~26.50, and
+find the potential difference of point $b$ relative to point $a$.
+\begin{center}
+\begin{verbatim}
+          1.00  20.0V  6.00
+      +---/\/\/--|i----/\/\/---+
+1.00A v   4.00     1.00   E1   |
+      a---/\/\/----/\/\/--|i---b
+2.00A v   1.00   E2    2.00    |
+      +---/\/\/--|i----/\/\/---+
+\end{verbatim}
+\end{center}
+\end{problem*}
+
+\begin{solution}
+First we'll simplify resistors in series, leading to the equivalent
+circuit
+\begin{center}
+\begin{verbatim}
+I_3=1.00A  20.0V  7.00=R_3
++---<-------|i----/\/\/---+
+|  I_1    R_1=5.00  E1    |
+a---<--------/\/\/--|i----b
+| I_2=2.00A E2    3.00=R_2|
++--->-------|i----/\/\/---+
+\end{verbatim}
+\end{center}
+We know everything about wire 3, so $V_{ba}$ is given by
+\begin{equation}
+  V_{ba} = -20.0\U{V} + I_3R_3
+    = -20.0\U{V} + 1.00\U{A}\cdot7.00\U{\Ohm}
+    = \ans{-13.0\U{V}}
+\end{equation}
+
+We also know everything about wire 2, so we can find $\EMF_2$ by
+looping clockwise from $a$ through 3 and 2.
+\begin{align}
+  0 &= V_{ba} + I_2R_2 + \EMF_2 \\
+  \EMF_2 &= -(V_{ba}+I_2R_2) = \ans{7.00\U{V}}
+\end{align}
+
+To find $\EMF_1$ we'll need $I_1$.  Applying Kirchoff's junction rule
+to $a$
+\begin{align}
+  0 &= I_3 + I_1 - I_2 \\
+  I_1 &= I_2 - I_3 = 2.00\U{A} - 1.00\U{A} = 1.00\U{A} \;.
+\end{align}
+Looping clockwise from $a$ through 3 and 1
+\begin{align}
+  0 &= V_{ba} + \EMF_1 - I_1R_1 \\
+  \EMF_1 &= I_1R_1 - V_{ba} = \ans{18.0\U{V}}
+\end{align}
+\end{solution}
diff --git a/latex/problems/Young_and_Freedman_12/problem26.48.tex b/latex/problems/Young_and_Freedman_12/problem26.48.tex
new file mode 100644 (file)
index 0000000..a75d8d4
--- /dev/null
@@ -0,0 +1,58 @@
+\begin{problem*}{26.48}
+In the circuit shown in Fig.~26.61, $C=5.90\U{$\mu$F}$, $\EMF =
+28.0\U{V}$, and the emf has negligible resistance.  Initially the
+capacitor is uncharged and the switch $S$ is in position 1, The switch
+is then moved to position 2, so that the capacitor begins to
+charge.  \Part{a} What will be the charge on the capacitor a long time
+after the switch is moved to position 2?  \Part{b} After the switch
+has been in position 2 for $3.00\U{ms}$, the charge on the capacitor
+is measured to be $110\U{$\mu$C}$.  What is the value of the
+resistance $R$?  \Part{c} How long after the switch is moved to
+position 2 will the charge on the capacitor be equal to $99.0\%$ of
+the final value found in \Part{a}.
+\begin{center}
+\begin{verbatim}
+ +---------.
+ |        /
+ |      S1  S2
+ |      |    |
+=== C   |   --- E
+ |      |    -
+ |      |    |
+ +-/\/\-+----+
+    R
+\end{verbatim}
+\end{center}
+\end{problem*}
+
+\begin{solution}
+\Part{a}
+After a long time in position 2, the capacitor will have become fully
+charged, no more current will flow through the circuit, and the
+voltage across the capacitor will balance that across the battery.
+\begin{equation}
+  Q = CV = CE = 5.90\U{$\mu$F}\cdot28.0\U{V} = \ans{165\U{$\mu$C}}
+\end{equation}
+
+\Part{b}
+Here we use the charging capacitor formula (Eqn.~26.12)
+\begin{align}
+  q &= CE\p({1-e^{-t/RC}}) \\
+  \frac{q}{CE} &= 1-e^{-t/RC} \\
+  e^{-t/RC} &= 1-\frac{q}{CE} \\
+  \frac{-t}{RC} &= \ln\p({1-\frac{q}{CE}}) \\
+  R &= \frac{-t}{C\ln\p({1-\frac{q}{CE}})}
+    = \frac{-3.00\U{ms}}
+           {5.90\U{$\mu$F}\cdot\ln\p({1-\frac{110\U{$\mu$C}}{165\U{$\mu$C}}})}
+    = \ans{463\U{\Ohm}}
+\end{align}
+
+\Part{c}
+Again, we use the charging-capacitor formula
+\begin{align}
+  \frac{-t}{RC} &= \ln\p({1-\frac{q}{CE}}) \\
+  t &= -RC\ln\p({1-\frac{q}{CE}})
+    = -463\U{\Ohm}\cdot5.90\U{$\mu$F}\cdot\ln(1-0.99)
+    = \ans{12.6\U{ms}}
+\end{align}
+\end{solution}